AMC 10A 2025

Problem 1

Andy and Betsy both live in Mathville. Andy leaves Mathville on his bicycle at $1: 30$ traveling due north at a steady 8 miles per hour. Betsy leaves on her bicycle from the same point at 2:30, traveling due east at a steady 12 miles per hour. At what time will they be exactly the same distance from their common starting point?
(A) $3: 30$
(B) $3: 45$
(C) $4: 00$
(D) $4: 15$
(E) $4: 30$

Solution 1

We can see that at $2: 30$, Andy will be 8 miles ahead. For every hour that they both travel, Betsy will gain 4 miles on Andy. Therefore, it will take 2 more hours for Betsy to catch up, and they will be at the same point at 4:30.

Problem 2

A box contains 10 pounds of a nut mix that is 50 percent peanuts, 20 percent cashews, and 30 percent almonds. A second nut mix containing 20 percent peanuts, 40 percent cashews, and 40 percent almonds is added to the box resulting in a new nut mix that is 40 percent peanuts. How many pounds of cashews are now in the box?
(A) 3.5
(B) 4
(C) 4.5
(D) 5
(E) 6

Solution 2

$$We are given $0.2(10)=2$ pounds of cashews in the first box.
Denote the pounds of nuts in the second nut mix as $x$.

$$
\begin{gathered}
5+0.2 x=0.4(10+x) \
0.2 x=1 \
x=5
\end{gathered}
$$

Thus, we have 5 pounds of the second mix.

$$
0.4(5)+2=2+2=4
$$

Problem 3

How many isosceles triangles are there with positive area whose side lengths are all positive integers and whose longest side has length $2025 ?$
(A) 2025
(B) 2026
(C) 3012
(D) 3037
(E) 4050

Solution 3

You can split the problem into two cases:
Case 1: The two sides with equal length are both smaller than 2025, which means that they range from 1013 to 2024. There are 1012 such cases.
Case 2 : There are two sides of length 2025 , so the last side must be in the range 1 to 2025 . There are 2025 such cases. Keep in mind, an equilateral triangle also counts as an isosceles triangle, since it has at least 2 sides of equal length.
Therefore, the total number of cases is 1012+2025= 3037

Problem 4

A team of students is going to compete against a team of teachers in a trivia contest. The total number of students and teachers is 15 . Ash, a cousin of one of the students, wants to join the contest. If Ash plays with the students, the average age on that team will increase from 12 to 14 . If Ash plays with the teachers, the average age on that team will decrease from 55 to 52 . How old is Ash?
(A) 28
(B) 29
(C) 30
(D) 32
(E) 33

Solution 4

When Ash joins a team, the team's average is pulled towards his age. Let $A$ be Ash's age and $N$ be the number of people on the student team. This means that there are $15-N$ people in the teacher team. Let us write an expression for the change in the average for each team.
The students originally had an average of 12 , which became 14 when Ash joined, so there was an increase of 2 . The term $A-12$ represents how much older Ash is compared to the average of the students'. If we divide this by $N+1$, which is the number of people on the student team when Ash joins, we get the average change per team member once Ash is added. Therefore,

$$
\frac{A-12}{N+1}=2 .
$$

Similarly, for teachers, the average was originally 55 , which decreased by 3 to become 52 when Ash joined. Intuitively, $55-A$ represents how much younger Ash is than the average age of the teachers. Dividing this by the expression $(15-N)+1$, which is the new total number of people on the teacher team, represents the average change per team member once Ash joins. We can write the equation

$$
\frac{55-A}{16-N}=3
$$

To solve the system, multiply equation (1) by $N+1$, and similarly multiply equation (2) by $16-N$. Then add the equations together, canceling $A$, leaving equation $43=50-N$. From this we get $N=7$ and A =28.

Problem 5

Consider the sequence of positive integers

$$
1,2,1,2,3,2,1,2,3,4,3,2,1,2,3,4,5,4,3,2,1,2,3,4,5,6,5,4,3,2,1,2, \ldots
$$

What is the 2025th term in this sequence?
(A) 5
(B) 15
(C) 16
(D) 44
(E) 45

Solution 5

One possible way the sequence could've been constructed was by putting "mountains" going up from 1 , to $n+1$, then going back down to 2 . For example, the first few "mountains" look like this:

$$
12|1232| 123432|12345432| \ldots
$$

So, the $n^{\text {th }}$ mountain has length $2 n$ and has highest number $n+1$. We want to add mountains until we get a total length as close as possible, but not exceeding, 2025. Let the last mountain we sum be mountain $a$. Hence,

$$
\begin{gathered}
2+4+6+\ldots+2 a=2(1+2+3+\ldots+a)=a(a+1) \leq 2025 \
\Longrightarrow a^2<2025 \Longrightarrow a<45
\end{gathered}
$$

so our max $a$ is 44 . In this $44^{\text {th }}$ mountain, the max number is 45 , so the $45^{\text {th }}$ mountain has max number 46 . Next, $44(44+1)=1980$, so we're looking for the $45^{\text {th }}$ number in the $45^{\text {th }}$ mountain, which is 45 .

Problem 6

In an equilateral triangle each interior angle is trisected by a pair of rays. The intersection of the interiors of the middle $20^{\circ}$-angle at each vertex is the interior of a convex hexagon. What is the degree measure of the smallest angle of this hexagon?
(A) 80
(B) 90
(C) 100
(D) 110
(E) 120

Solution 6

Assume you have a diagram in front of you.
Because each angle of the triangle is trisected, we have $920^{\circ}$ angles. Using a side of the triangle as a base, we have an isosceles triangle with two $20^{\circ}$ angles. Using this we can show that the third angle is $140^{\circ}$.
Following that, we use the principle of vertical angles to show that one angle of the hexagon is $140^{\circ}$. And with rotational symmetry, three. The average of all 6 angles has to be $120^{\circ}$, so the answer is 100

Problem 7

Suppose $a$ and $b$ are real numbers. When the polynomial $x^3+x^2+a x+b$ is divided by $x-1$, the remainder is 4 . When the polynomial is divided by $x-2$, the remainder is 6 . What is $b-a$ ?
(A) 14
(B) 15
(C) 16
(D) 17
(E) 18

Solution 7

Use synthetic division to find that the remainder of $x^3+x^2+a x+b$ is $a+b+2$ when divided by $x-1$ and $2 a+b+12$ when divided by $x-2$. Now, we solve

$$
\left{\begin{array}{l}
a+b+2=4 \
2 a+b+12=6
\end{array}\right.
$$

This ends up being $a=-8, b=10$, so $b-a=10-(-8)= 18

Problem 8

Agnes writes the following four statements on a blank piece of paper.

Each statement is either true or false. How many false statements did Agnes write on the paper?
(A) 0
(B) 1
(C) 2
(D) 3
(E) 4

Solution 8

We first number all the statements:
1) At least one of these statements is true. 2) At least two of these statements are true. 3) At least two of these statements are false. 4) At least one of these statements is false.

We can immediately see that statement 4 must be true, as it would contradict itself if it were false. Similarly, statement 1 must be true, as all the other statements must be false if it were false, which is contradictory because statement 4 is true. Since both 1 and 4 are true, statement 2 has to be true. Therefore, statement 3 is the only false statement, making the answer 1.

Problem 9

Let $f(x)=100 x^3-300 x^2+200 x$. For how many real numbers $a$ does the graph of $y=f(x-a)$ pass through the point $(1,25)$ ?
(A) 1
(B) 2
(C) 3
(D) 4
(E) more than 4

Solution 9

Substitute $1-a$ for $x$ and set this expression equal to 25 . The problem boils down to finding how many real roots

$$
100(1-a)^3-300(1-a)^2+200(1-a)=25
$$

has. We further simplify this expression and create a function $f(x)$ :

$$
f(x)=-100 a^3+100 a-25
$$

Using Descarte's Rule of Signs we get:
Sign changes for $f(x)$ (possible number of positive roots): 2

$$
f(-x)=+100 a^3-100 a-25
$$

Sign changes for $f(-x)$ (possible number of negative roots): 1

Possibilities for roots:
1) 2 positive roots, 1 negative root
2) 0 positive roots, 1 negative root, 2 imaginary roots

So which one is it? We know if the function changes sign between an interval, then a root exists in that interval. From $a=0$ to $\frac{1}{2}$, the function changes sign because $f(0)=-25$ while $f\left(\frac{1}{2}\right)=+\frac{25}{2}$, so a positive root exists. This eliminates the second possibility, implying that there must be 2 positive and 1 negative roots. So the answer is $2+1= 3$.

Problem 10

A semicircle has diameter $\overline{A B}$ and chord $\overline{C D}$ of length 16 parallel to $\overline{A B}$. A smaller semicircle with diameter on $\overline{A B}$ and tangent to $\overline{C D}$ is cut from the larger semicircle, as shown below.

What is the area of the resulting figure, shown shaded?
(A) $16 \pi$
(B) $24 \pi$
(C) $32 \pi$
(D) $48 \pi$
(E) $64 \pi$

Solution 10

The problem doesn't restrict where the smaller semicircle is along the larger semicircle's diameter. Therefore, we can assume that the two semicircles are concentric. Let the center of both semicircles be $O$, and let $C D$ be tangent to the smaller semicircle at $T$. Let the radius of the smaller semicircle be $x$, and let the radius of the larger semicircle be $r$. If we mirror the diagram over $A B$, we can see that we have two concentric circles. We are trying to find $\pi\left(\frac{r^2-x^2}{2}\right)$. By Power of a Point on $T$, we can see that

$$
64=(r+x)(r-x)=r^2-x^2
$$

Thus, $\pi\left(\frac{r^2-x^2}{2}\right)=32$

Problem 11

The sequence $1, x, y, z$ is arithmetic. The sequence $1, p, q, z$ is geometric. Both sequences are strictly increasing and contain only integers, and $z$ is as small as possible. What is the value of $x+y+z+p+q$ ?
(A) 66
(B) 91
(C) 103
(D) 132
(E) 149

Solution 11

Since $1, x, y, z$ is an arithmetic sequence, we have $y=2 x-1$ and $z=3 x-2$. Since $1, p, q, z$ is a geometric sequence, we have $q=p^2$ and $z=p^3$. Thus $p^3=3 x-2$.

Because $p^3 \equiv p(\bmod 3)$, we get $3 x-2 \equiv p(\bmod 3)$, so $p \equiv 1(\bmod 3)$. The smallest integer $p>1$ satisfying this is $p=4$.
Then $64=3 x-2 \rightarrow x=22, y=43, z=64, q=16$. Therefore, $x+y+z+p+q=22+43+64+4+16= 149$.

Problem 12

Carlos uses a 4-digit passcode to unlock his computer. In his passcode, exactly one digit is even, exactly one (possibly different) digit is prime, and no digit is 0 . How many 4 -digit passcodes satisfy these conditions?
(A) 176
(B) 192
(C) 432
(D) 464
(E) 608

Solution 12

The only two digits that are neither prime nor even are 1 and 9 . We split this problem into cases based on the number of 2 s . This is because 2 is both a prime number and an even number.

Case 1: For this case, there are no 2s. For this case, there are 4 choices for where the even digit goes, and 3 choices for what the even digit is. There are then 3 choices for where the prime digit goes, and 3 choices for what the prime digit is. The last two spots have 2 choices each, 1 or 9 . This gives a total of $4 \cdot 3^3 \cdot 2^2=432$ options for this case.
Case 2: For this case, there is one 2 . There are 4 choices for where 2 goes, and 2 choices for the other three digits each. This case gives a total of $2^3 \cdot 4=32$ options!
Hence, the answer is $432+32=464$

Problem 13

In the figure below, the outside square contains infinitely many squares, each of them with the same center and sides parallel to the outside square. The ratio of the side length of a square to the side length of the next inner square is $k$, where $0<k<1$. The spaces between squares are alternately shaded as shown in the figure (which is not necessarily drawn to scale).

The area of the shaded portion of the figure is $64 \%$ of the area of the original square. What is $k$ ?
(A) $\frac{3}{5}$
(B) $\frac{16}{25}$
(C) $\frac{2}{3}$
(D) $\frac{3}{4}$
(E) $\frac{4}{5}$

Solution 13

Let the side length of the largest square be $a$, so it has area $a^2$. Hence, the second-largest square has area $a^2 k^2$, the third-largest has $a^2 k^4$, and so on.

It follows that the total shaded area is

$$
a^2-a^2 k^2+a^2 k^4-a^2 k^6+\ldots=a^2\left(1-k^2+k^4-k^6+\ldots\right)=a^2 \frac{1}{1+k^2}
$$

The ratio of the area of the shaded region to that of the original square is then

$$
\begin{gathered}
\frac{a^2 \frac{1}{1+k^2}}{a^2}=\frac{1}{1+k^2}=\frac{64}{100} \
\Longrightarrow 64+64 k^2=100 \Longrightarrow k^2=\frac{36}{64} \Longrightarrow k= \frac{3}{4} .
\end{gathered}
$$

Problem 14

Six chairs are arranged around a round table. Two students and two teachers randomly select four of the chairs to sit in. What is the probability that the two students will sit in two adjacent chairs and the two teachers will also sit in two adjacent chairs?
(A) $\frac{1}{6}$
(B) $\frac{1}{5}$
(C) $\frac{2}{9}$
(D) $\frac{3}{13}$
(E) $\frac{1}{4}$

Solution 14

Pair two students together and put them adjacent on any two seats. There are 6 ways to do this. Considering one of these cases (they are all the same), there are 4 seats left, in which we wish to arrange the teachers together. So pair the teachers together and put them adjecent on any two seats not already occupied by two of the students. There are 3 ways to do this. For all 6 cases, there are $6 \times 3=18$ favorable outcomes.
The number of ways to arrange the 2 students and 2 teachers is $\binom{6}{2} \times\binom{ 4}{2}=90$.
Our probability is $\frac{18}{90}={5}$

Problem 15

In the figure below, $A B E F$ is a rectangle, $\overline{A D} \perp \overline{D E}, A F=7, A B=1$, and $A D=5$.

What is the area of $\triangle A B C ?$
(A) $\frac{3}{8}$
(B) $\frac{4}{9}$
(C) $\frac{1}{8} \sqrt{13}$
(D) $\frac{7}{15}$
(E) $\frac{1}{8} \sqrt{15}$

Solution 15

Because $A B E F$ is a rectangle, $\angle A B C=90$. We are given that $\angle A D E=90$, and since $\angle E C D=\angle A C B$ by vertical angles, $\triangle E C D \sim \triangle A C B$. Let $A C=x$. By the Pythagorean Theorem, $C B=\sqrt{x^2-1}$. Since $A F=B E=7, E C=7-\sqrt{x^2-1}$. Because $A C=x$ and $A D=5, C D=5-x$. By similar triangles,

$$
\frac{7-\sqrt{x^2-1}}{x}=\frac{5-x}{\sqrt{x^2-1}}
$$

. Cross-multiplying, we get that

$$
7 \sqrt{x^2-1}-x^2+1=5 x-x^2
$$

, so

$$
7 \sqrt{x^2-1}=5 x-1
$$

. We square both sides, and this is simply a quadratic in $x$ :

$$
24 x^2+10 x-50=0
$$

, which has a positive root $x=\frac{5}{4}$. Since $A B=1$, we can plug this into the Pythagorean Theorem, with $\frac{5}{4}$ being the hypotenuse, to get $B C=\frac{3}{4}$, and $1 \cdot \frac{\frac{3}{4}}{2}$ to equal $[A B C]= \frac{3}{8}$

Problem 16

There are three jars. Each of three coins is placed in one of the three jars, chosen at random and independently of the placements of the other coins. What is the expected number of coins in a jar with the most coins?
(A) $\frac{4}{3}$
(B) $\frac{13}{9}$
(C) $\frac{5}{3}$
(D) $\frac{17}{9}$
(E) 2

Solution 16

We have three coins and three jars. Each coin is placed independently and randomly into one of the jars. Let $M$ be the maximum number of coins in any jar. We want to compute the expected value of $M$.

Step 1: Count total outcomes
Each coin has 3 choices, so the total number of equally likely placements is $3^3=27$.
Step 2: Casework on the maximum number of coins
Case 1: $M=1$. This occurs when each jar has exactly one coin. There are $3!=6$ assignments of coins to jars. Hence, $\operatorname{Pr}(M=1)=\frac{6}{27}=\frac{2}{9}$.
Case 2: $M=3$. This occurs when all three coins fall into the same jar. There are 3 jars to choose from, so $\operatorname{Pr}(M=3)=\frac{3}{27}=\frac{1}{9}$.
Case 3: $M=2$. This occurs when one jar has 2 coins, another jar has 1 coin, and the last jar has 0 coins. We can choose which jar gets 2 coins in 3 ways, which jar gets 1 coin in 2 ways, and which 2 coins out of the 3 go into the jar with two coins, so we multiply by $\binom{3}{2}$, which is just 3 (note we don't have to do this for the earlier cases because for case 2 , all 3 coins go into one jar, and for case 1 , the factorial already accounts for that). Therefore, there are $3^2 \cdot 2=18$ outcomes. Thus, $\operatorname{Pr}(M=2)=\frac{18}{27}=\frac{2}{3}$.
Step 3: Compute the expected value The expected value of $M$ is $\mathbb{E}[M]=1 \cdot \frac{2}{9}+2 \cdot \frac{2}{3}+3 \cdot \frac{1}{9}$. Converting everything to ninths, we have $\mathbb{E}[M]=\frac{2}{9}+\frac{12}{9}+\frac{3}{9}=\frac{17}{9}$.
Hence, the expected number of coins in the jar with the most coins is $\frac{17}{9}$.

Problem 17

Let $N$ be the unique positive integer such that dividing 273436 by $N$ leaves a remainder of 16 and dividing 272760 by $N$ leaves a remainder of 15 . What is the tens digit of $N$ ?
(A) 0
(B) 1
(C) 2
(D) 3
(E) 4

Solution 17

The problem statement implies that $N$ divides both $273436-16=273420$ and $272760-15=272745$. We want to find $N>16$ that satisfies both of these conditions. Hence, we can just find the greatest common divisor of the two numbers. $\operatorname{gcd}(273420,272745)=\operatorname{gcd}(675,272745)=\operatorname{gcd}(675,45)=45$ by the Euclidean Algorithm, so the answer is 4.

Problem 18

The harmonic mean of a collection of numbers is the reciprocal of the arithmetic mean of the reciprocals of the numbers in the collection. For example, the harmonic mean of 4,4 , and 5 is

$$
\frac{1}{\frac{1}{3}\left(\frac{1}{4}+\frac{1}{4}+\frac{1}{5}\right)}=\frac{30}{7}
$$

What is the harmonic mean of all the real roots of the 4050th degree polynomial

$$
\prod_{k=1}^{2025}\left(k x^2-4 x-3\right)=\left(x^2-4 x-3\right)\left(2 x^2-4 x-3\right)\left(3 x^2-4 x-3\right) \ldots\left(2025 x^2-4 x-3\right) ?
$$

(A) $-\frac{5}{3}$
(B) $-\frac{3}{2}$
(C) $-\frac{6}{5}$
(D) $-\frac{5}{6}$
(E) $-\frac{2}{3}$

Solution 18

We will need to determine the sum of the reciprocals of the roots. To find the sum of the reciprocals of the roots $p, q$ of the quadratic $a x^2+b x+c$, we use Vieta's formulas. Recall that $p+q=-b / a$ and $p q=c / a$. Therefore,

$$
\frac{1}{p}+\frac{1}{q}=\frac{p+q}{p q}=\frac{\frac{-b}{a}}{\frac{c}{a}}=\frac{-b}{a} \cdot \frac{a}{c}=\frac{-b}{c}
$$

which doesn't depend on $a$.
The sum of the reciprocals of the roots of the quadratic $x^2-4 x-3$ is $\frac{-(-4)}{-3}=-4 / 3$. The same is true for every quadratic in the form $a x^2-4 x-3$. The sum of all the reciprocals of the roots of $a x^2+b x+c$ is $2025 \cdot\left(-\frac{4}{3}\right)$.
Because we have 2025 quadratics, there are $2 \cdot 2025=4050$ total roots. Our answer is $\frac{1}{\frac{1}{4050} \cdot \frac{-4 \cdot 2025}{3}}=-\frac{3}{2}$.

Problem 19

An array of numbers is constructed beginning with the numbers $-1 \quad 3 \quad 1$ in the top row. Each adjacent pair of numbers is summed to produce a number in the next row. Each row begins and ends with -1 and 1 , respectively.

$$
\begin{array}{cccccccc}
& & -1 & & 3 & & 1 & \
& & & & & & & \
-1 & & 2 & & 4 & & 1 & \
& & & & & & & \
-1 & 1 & & 6 & & 5 & & 1
\end{array}
$$

If the process continues, one of the rows will sum to 12,288 . In that row, what is the third number from the left?
(A) -29
(B) -21
(C) -14
(D) -8
(E) -3

Solution 19

Consider the polynomial $f(x)=-x^2+3 x+1$. When we multiply this polynomial by $x+1$, we are essentially doing the operation given in the problem (When we multiply $p(x)$ by $x+1$, a term of degree $d$ in the yielded expression is the sum of $1 \cdot$ (degree d ) and $x \cdot$ (degree $\mathrm{d}-1$ ) in $p(x)$ This effect is visible in Pascal's Triangle). So, if we let the coefficients of $f(x)$ be the zero row of the array, then the $n^{\text {th }}$ row is just the coefficients of $f(x)(x+1)^n$. The next thing to note is that the sum of the coefficients in any polynomial $p(x)$ is just $p(1)$. Therefore, the sum of the entries in the $n^{\text {th }}$ row of the array is $f(1)(1+1)^n=3 \cdot 2^n$. Letting this equal 12288 , we get $n=12$. We are looking for the $3^{\text {rd }}$ term in the $12^{\text {th }}$ row. The $12^{\text {th }}$ row is given by the coefficients of $f(x)(x+1)^{12}=\left(-x^2+3 x+1\right)(x+1)^{12}$. Since the degree of the resulting expression is 14 , the third term in the row is just the coefficient of $x^{12}$ in the expression, which is $-\binom{12}{10}+3\binom{12}{11}+1=-29$.

Problem 20

A silo (right circular cylinder) with diameter 20 meters stands in a field. MacDonald is located 20 meters west and 15 meters south of the center of the silo. McGregor is located 20 meters east and $g>0$ meters south of the center of the silo. The line of sight between MacDonald and McGregor is tangent to the silo. The value of $g$ can be written as $\frac{a \sqrt{b}-c}{d}$, where $a, b, c$, and $d$ are positive integers, $b$ is not divisible by the square of any prime, and $d$ is relatively prime to the greatest common divisor of $a$ and $c$. What is $a+b+c+d$ ?
(A) 119
(B) 120
(C) 121
(D) 122
(E) 123

Solution 20

Let the silo center be $O$, let the point MacDonald is situated at be $A$, and let the point 20 meters west of the silo center be $B . A B O$ is then a right triangle with side lengths 15,20 , and 25 .

Let the point 20 meters east of the silo center be $C$, and let the point McGregor is at be $D$ with $C D=g>0$. Also let $A D$ be tangent to circle $O$ at $E$.

Extend $B C$ and $A D$ to meet at point $F$. This creates 3 similar triangles, $\triangle A B F \sim \triangle D C F \sim \triangle O E F$. Let the distance between point $C$ and $F$ be $x$. The similarity ratio between triangles $A B F$ and $D C F$ is then $\frac{\text { longer leg }}{\text { shorter leg }}=\frac{40+x}{15}=\frac{x}{g}$

This is currently unsolvable so we bring in triangle $O E F$. The hypotenuse of triangle $O E F$ is $O F=20+x$ and its shorter leg is the radius of the silo $=10$. We can then establish a second similarity relationship between triangles $O E F$ and $A B F$ with $\frac{\text { shorter leg }}{\text { hypotenuse }}=\frac{10}{20+x}=\frac{15}{A F}$

Now we find the hypotenuse of $A B F$ in terms of $x$ using the Pythagorean theorem. $A F^2=15^2+(40+x)^2$. Which simplifies to $A F^2=225+1600+80 x+x^2=1825+80 x+x^2$ So $A F=\sqrt{x^2+80 x+1825}$

Plugging back in we get $\frac{10}{20+x}=\frac{15}{\sqrt{x^2+80 x+1825}}$. Now we can begin to break this down by multiplying both sides by both denominators. $10\left(\sqrt{x^2+80 x+1825}\right)=15(20+x)$ Dividing both sides by 5 then squaring yields, $4 x^2+320 x+7300=9 x^2+360 x+3600$ This furthermore simplifies to $5 x^2+40 x-3700=0$ At which point we can divide off a 5 and then apply the quadratic formula on $x^2+8 x-740=0$ which we take the positive root of.

$$
x=\frac{-8+\sqrt{64+2960}}{2}=\frac{-8+\sqrt{3024}}{2}=\frac{-8+\sqrt{144 \cdot 21}}{2} .
$$

Simplifying yields that $x=6 \sqrt{21}-4$

Then to solve for $g$ we simply plug $6 \sqrt{21}-4$ back into the first similarity ratio to get $\frac{36+6 \sqrt{21}}{15}=\frac{6 \sqrt{21}-4}{g}$

Multiply both sides by $15 g$ and dividing by $36+6 \sqrt{21}$ will let us solve for $g=\frac{15(6 \sqrt{21}-4)}{36+6 \sqrt{21}}$ and after rationalizing the denominator we get $\frac{20 \sqrt{21}-75}{3} \cdot 20+21+75+3=119$

Problem 21

A set of numbers is called sum-free if whenever $x$ and $y$ are (not necessarily distinct) elements of the set, $x+y$ is not an element of the set. For example, ${1,4,6}$ and the empty set are sum-free, but ${2,4,5}$ is not. What is the greatest possible number of elements in a sum-free subset of ${1,2,3, \ldots, 20}$ ?
(A) 8
(B) 9
(C) 10
(D) 11
(E) 12

Solution 21

Let our subset be ${11,12,13, \ldots, 20}$. If we add any element from the set ${1,2,3, \ldots, 10}$ to our current subset, we will have to remove at least one element from our subset. Hence, the maximum size of our subset is 10 .

Problem 22

A circle of radius $r$ is surrounded by three circles, whose radii are 1,2 , and 3 , all externally tangent to the inner circle and externally tangent to each other, as shown in the diagram below.

What is $r$ ?
(A) $\frac{1}{4}$
(B) $\frac{6}{23}$
(C) $\frac{3}{11}$
(D) $\frac{5}{17}$
(E) $\frac{3}{10}$

Solution 22

Descartes' Circle Formula (curvatures $k_i=\frac{1}{r_i}$ )

$$
k_4=k_1+k_2+k_3 \pm 2 \sqrt{k_1 k_2+k_2 k_3+k_3 k_1}
$$

For radii $1,2,3$ we have

$$
k_1=1, \quad k_2=\frac{1}{2}, \quad k_3=\frac{1}{3}
$$

Compute the sum and the square-root term

$$
k_1+k_2+k_3=\frac{11}{6}, \quad k_1 k_2+k_2 k_3+k_3 k_1=1
$$

Therefore

$$
k_4=\frac{11}{6} \pm 2
$$

Choose the plus sign for the small circle tangent externally to the three given circles

$$
k_4=\frac{11}{6}+2=\frac{23}{6}, \quad r_4=\frac{1}{k_4}=\frac{6}{23}
$$

Problem 23

Triangle $\triangle A B C$ has side lengths $A B=80, B C=45$, and $A C=75$. The bisector of $\angle B$ and the altitude to side $\overline{A B}$ intersect at point $P$. What is $B P$ ?
(A) 18
(B) 19
(C) 20
(D) 21
(E) 22

Solution 23

Let $C D \perp A B$ with foot $D$. Right triangles $A C D$ and $B C D$ give $A C^2=A D^2+C D^2, B C^2=B D^2+C D^2$,

$$
A C^2-B C^2=A D^2-B D^2=(A D-B D)(A D+B D)
$$

Since $A D+B D=A B=80$ and $A C^2-B C^2=75^2-45^2=3600$, we get the equation $3600=80(A D-B D)$. This equation simplifies to $45=A D-B D$. We can solve the system of equations $A D+B D=80$ and $A D-B D=45$ easily via elimination, and we

$$
\text { get } A D=\frac{125}{2}, B D=\frac{35}{2} . C D^2=A C^2-A D^2=75^2-\left(\frac{125}{2}\right)^2=\frac{6875}{4}, C D=\frac{25 \sqrt{11}}{2} \text {. }
$$

By Angle Bisector Theorem, $\frac{D P}{P C}=\frac{D B}{B C}=\frac{\frac{35}{2}}{45}=\frac{7}{18}, P C=C D-D P$ thus, $18 D P=7(C D-D P), 25 D P=7 C D$,

$$
\begin{aligned}
& D P=\left(\frac{7}{25}\right) C D=\left(\frac{7}{25}\right)\left(\frac{25 \sqrt{11}}{2}\right)=\frac{7 \sqrt{11}}{2} \
& B P^2=B D^2+D P^2=\left(\frac{35}{2}\right)^2+\left(\frac{7 \sqrt{11}}{2}\right)^2=\frac{1225}{4}+\frac{49(11)}{4}=\frac{1764}{4}=441, \text { thus } B P= 21 .
\end{aligned}
$$

Problem 24

Call a positive integer fair if no digit is used more than once, it has no 0s, and no digit is adjacent to two greater digits. For example, 196, 23, and 12463 are fair, but 1546,320 , and 34321 are not fair. How many fair positive integers are there?
(A) 511
(B) 2584
(C) 9841
(D) 17711
(E) 19682

Solution 24

Note every fair number will have an increasing string of digits, a maximum digit, then a decreasing string of digits. This is because if it decreases then increases, then the digit in the middle will be less than its adjacent digits.

Let $n$ be the maximum digit. For each number $i<n$, we may either place $i$ before $n$, after $n$, or choose not to include it. Note this process will result in a unique number for every case, as the numbers before $n$ must be in increasing order, and the numbers after $n$ must be in decreasing order. Therefore, for each number $n$, we have $3^{n-1}$ cases.
Since $n \in{1,2, \cdots 9}$, we have:

$$
\sum_{n=1}^9 3^{n-1}=\frac{3^9-1}{3-1}=9841
$$

Problem 25

A point $P$ is chosen at random inside square $A B C D$. The probability that $\overline{A P}$ is neither the shortest nor the longest side of $\triangle A P B$ can be written as $\frac{a+b \pi-c \sqrt{d}}{e}$, where $a, b, c, d$, and $e$ are positive integers, $\operatorname{gcd}(a, b, c, e)=1$, and $d$ is not divisible by the square of a prime. What is $a+b+c+d+e$ ?
(A) 25
(B) 26
(C) 27
(D) 28
(E) 29

Solution 25

Assume the sides of this square is 1 , hence we only need to find the area of the desired regions. From Solution 1, it is easy to see that the regions are the bottom left region c and the top right region b , hence we must compute $b+c$. Also, define $k$ to be the circular segment. We have two equations right off the bat:
$2 a+2 b+2 c=1 \Longrightarrow a+b+c=\frac{1}{2}$ since the sum of all regions is just the area of the square and also, $2 b+a=\frac{\pi}{4}$, just the area of a quarter-circle.
Next, $\triangle A B N$ has a area of $\frac{\sqrt{3}}{4}$ since it is just an equilateral triangle with length 1 (each side is a radius of a circle with radius of 1 ). From the diagram, $2 k+[A B N]=2 k+\frac{\sqrt{3}}{4}=2 b$. Subsequently, we see that sector $A D N$ has an angle of $90-60=30$ and is the sum of $a+k$. Therefore, $a+k=\frac{\pi}{12}$.
Multiply this equation by 2 : $2 a+2 k=\frac{\pi}{6}$ and combining it with $2 b-2 k=\frac{\sqrt{3}}{4}$ yields $2 a+2 b=\frac{\pi}{6}+\frac{\sqrt{3}}{4}$. Since we also have that $2 b+a=\frac{\pi}{4}$, subtracting this from the equation yields $a=\left(\frac{\pi}{6}+\frac{\sqrt{3}}{4}\right)-\left(\frac{\pi}{4}\right)=\frac{-\pi}{12}+\frac{\sqrt{3}}{4}$. We are to find $b+c=\frac{1}{2}-a=\frac{1}{2}-\left(\frac{\sqrt{3}}{4}-\frac{\pi}{12}\right)=\frac{1}{2}-\frac{\sqrt{3}}{4}+\frac{\pi}{12}=\frac{6+\pi-3 \sqrt{3}}{12}$.
At last, $a+b+c+d+e=6+1+3+3+12= 25$.

American Math Competition (AMC) 10 B - Problem and Solution

Problem 1

In a long line of people arranged left to right, the 1013th person from the left is also the 1010th person from the right. How many people are in line?

(A) 2021
(B) 2022
(C) 2023
(D) 2024
(E) 2025

Problem 2

What is $10!-7!\cdot 6!$

(A) -120
(B) 0
(C) 120
(D) 600
(E) 720

Problem 3

For how many integer values of $x$ is $|2 x| \leq 7 \pi$

(A) 16
(B) 17
(C) 19
(D) 20
(E) 21

Problem 4

Balls numbered $1,2,3, \ldots$ are deposited in 5 bins, labeled $A, B, C, D$, and $E$, using the following procedure. Ball 1 is deposited in bin $A$, and balls 2 and 3 are deposited in bin $B$. The next 3 balls are deposited in bin $C$, the next 4 in bin $D$, and so on, cycling back to bin $A$ after balls are deposited in bin E. (For example, balls numbered $22,23, \ldots, 28$ are deposited in bin B at step 7 of this process.) In which bin is ball 2024 deposited?

(A) $A$
(B) $B$
(C) $C$
(D) $D$
(E) $E$

Problem 5

In the following expression, Melanie changed some of the plus signs to minus signs:

$$
1+3+5+7+\ldots+97+99
$$

When the new expression was evaluated, it was negative. What is the least number of plus signs that Melanie could have changed to minus signs?

(A) 14
(B) 15
(C) 16
(D) 17
(E) 18

Problem 6

A rectangle has integer length sides and an area of 2024. What is the least possible perimeter of the rectangle?

(A) 160
(B) 180
(C) 16
(D) 17
(E) 18

Problem 7

What is the remainder when $7^{2024}+7^{2025}+7^{2026}$ is divided by $19 ?$

(A) 0
(B) 1
(C) 7
(D) 11
(E) 18

Problem 8

Let $N$ be the product of all the positive integer divisors of 42 . What is the units digit of $N$ ?

(A) 0
(B) 2
(C) 4
(D) 6
(E) 8

Problem 9

Real numbers $a, b$, and $c$ have arithmetic mean 0 . The arithmetic mean of $a^2, b^2$, and $c^2$ is 10 . What is the arithmetic mean of $a b, a c$, and $b c$ ?

(A) -5
(B) $-\frac{10}{3}$
(C) $-\frac{10}{9}$
(D) 0
(E) $\frac{10}{9}$

Problem 10

Quadrilateral $A B C D$ is a parallelogram, and $E$ is the midpoint of the side $A D$. Let $F$ be the intersection of lines $E B$ and $A C$. What is the ratio of the area of quadrilateral $C D E F$ to the area of triangle $C F B$ ?

(A) $5: 4$
(B) $4: 3$
(C) $3: 2$
(D) $5: 3$
(E) $2: 1$

Problem 11

In the figure below $W X Y Z$ is a rectangle with $W X=4$ and $W Z=8$. Point $M$ lies $\overline{X Y}$, point $A$ lies on $\overline{Y Z}$, and $\angle W M A$ is a right angle. The areas of $\triangle W X M$ and $\triangle W A Z$ are equal. What is the area of $\triangle W M A$ ?

(A) 13
(B) 14
(C) 15
(D) 16
(E) 17

Problem 12

A group of 100 students from different countries meet at a mathematics competition. Each student speaks the same number of languages, and, for every pair of students $A$ and $B$, student $A$ speaks some language that student $B$ does not speak, and student $B$ speaks some language that student $A$ does not speak. What is the least possible total number of languages spoken by all the students?

(A) 9
(B) 10
(C) 12
(D) 51
(E) 100

Problem 13

Positive integers $x$ and $y$ satisfy the equation $\sqrt{x}+\sqrt{y}=\sqrt{1183}$. What is the minimum possible value of $x+y$.

(A) 585
(B) 595
(C) 623
(D) 700
(E) 791

Problem 14

A dartboard is the region B in the coordinate plane consisting of points $(x, y)$ such that $|x|+|y| \leq 8$. A target T is the region where $\left(x^2+y^2-25\right)^2 \leq 49$. A dart is thrown at a random point in B. The probability that the dart lands in $T$ can be expressed as $\frac{m}{n} \pi$, where $m$ and $n$ are relatively prime positive integers. What is $m+n$ ?

(A) 39
(B) 71
(C) 73
(D) 75
(E) 135

Problem 15

A list of 9 real numbers consists of $1,2.2,3.2,5.2,6.2,7$, as well as $x, y, z$ with $x \leq y \leq z$. The range of the list is 7 , and the mean and median are both positive integers. How many ordered triples $(x, y, z)$ are possible?

(A) 1
(B) 2
(C) 3
(D) 4
(E) infinitely many

Problem 16

Jerry likes to play with numbers. One day, he wrote all the integers from 1 to 2024 on the whiteboard. Then he repeatedly chose four numbers on the whiteboard, erased them, and replaced them with either their sum or their product. (For example, Jerry's first step might have been to erase $1,2,3$, and 5 , and then write either 11 , their sum, or 30 , their product, on the whiteboard.) After repeatedly performing this operation, Jerry noticed that all the remaining numbers on the board were odd. What is the maximum possible number of integers on the board at that time?

(A) 1010
(B) 1011
(C) 1012
(D) 1013
(E) 1014

Problem 17

In a race among 5 snails, there is at most one tie, but that tie can involve any number of snails. For example, the result might be that Dazzler is first; Abby, Cyrus, and Elroy are tied for second; and Bruna is fifth. How many different results of the race are possible?

(A) 180
(B) 361
(C) 420
(D) 431
(E) 720

Problem 18

How many different remainders can result when the 100th power of an integer is divided by 125?

(A) 1
(B) 2
(C) 5
(D) 25
(E) 125

Problem 19

In the following table, each question mark is to be replaced by "Possible" or "Not Possible" to indicate whether a nonvertical line with the given slope can contain the given number of lattice points (points both of whose coordinates are integers). How many of the 12 entries will be "Possible"?

(A) 4
(B) 5
(C) 6
(D) 7
(E) 9

Problem 20

Three different pairs of shoes are placed in a row so that no left shoe is next to a right shoe from a different pair. In how many ways can these six shoes be lined up?

(A) 60
(B) 72
(C) 90
(D) 108
(E) 120

Problem 21

Two straight pipes (circular cylinders), with radii 1 and $\frac{1}{4}$, lie parallel and in contact on a flat floor. The figure below shows a head-on view. What is the sum of the possible radii of a third parallel pipe lying on the same floor and in contact with both?

(A) $\frac{1}{9}$
(B) 1
(C) $\frac{10}{9}$
(D) $\frac{11}{9}$
(E) $\frac{19}{9}$

Problem 22

A group of 16 people will be partitioned into 4 indistinguishable 4-person committees. Each committee will have one chairperson and one secretary. The number of different ways to make these assignments can be written as $3^r M$, where $r$ and $M$ are positive integers and $M$ is not divisible by 3 . What is $r$ ?

(A) 5
(B) 6
(C) 7
(D) 8
(E) 9

Problem 23

The Fibonacci numbers are defined by $F_1=1, F_2=1$, and $F_n=F_{n-1}+F_{n-2}$ for $n \geq 3$. What is

$$
\frac{F_2}{F_1}+\frac{F_4}{F_2}+\frac{F_6}{F_3}+\ldots+\frac{F_{20}}{F_{10}} ?
$$

(A) 318
(B) 319
(C) 320
(D) 321
(E) 322

Problem 24

Let

$$
P(m)=\frac{m}{2}+\frac{m^2}{4}+\frac{m^4}{8}+\frac{m^8}{8}
$$

How many of the values $P(2022), P(2023), P(2024)$, and $P(2025)$ are integers?

(A) 0
(B) 1
(C) 2
(D) 3
(E) 4

Problem 25

Each of 27 bricks (right rectangular prisms) has dimensions $a \times b \times c$, where $a, b$ , and $c$ are pairwise relatively prime positive integers. These bricks are arranged to form a $3 \times 3 \times 3$ block, as shown on the left below. A $28^{\text {th }}$ brick with the same dimensions is introduced, and these bricks are reconfigured into a $2 \times 2 \times 7$ block, shown on the right. The new block is 1 unit taller, 1 unit wider, and 1 unit deeper than the old one. What is $a+b+c$ ?

(A) 88
(B) 89
(C) 90
(D) 91
(E) 92

American Math Competition (AMC) 10 A 2024 - Problem and Solution

Problem 1

What is the value of $9901 \cdot 101-99 \cdot 10101 ?$

(A) 2
(B) 20
(C) 200
(D) 202
(E) 2020

Problem 2

A model used to estimate the time it will take to hike to the top of the mountain on a trail is of the form $T=a L+b G$, where $a$ and $b$ are constants, $T$ is the time in minutes, $L$ is the length of the trail in miles, and $G$ is the altitude gain in feet. The model estimates that it will take 69 minutes to hike to the top if a trail is 1.5 miles long and ascends 800 feet, as well as if a trail is 1.2 miles long and ascends 1100 feet. How many minutes does the model estimates it will take to hike to the top if the trail is 4.2 miles long and ascends 4000 feet?

(A) 240
(B) 246
(C) 252
(D) 258
(E) 264

Problem 3

What is the sum of the digits of the smallest prime that can be written as a sum of 5 distinct primes?

(A) 5
(B) 7
(C) 9
(D) 10
(E) 13

Problem 4

The number 2024 is written as the sum of not necessarily distinct two-digit numbers. What is the least number of two-digit numbers needed to write this sum?

(A) 20
(B) 21
(C) 22
(D) 23
(E) 24

Problem 5

What is the least value of $n$ such that $n$ ! is a multiple of 2024 ?

(A) 11
(B) 21
(C) 22
(D) 23
(E) 253

Problem 6

What is the minimum number of successive swaps of adjacent letters in the string $A B C D E F$ that are needed to change the string to $F E D C B A$ ? (For example, 3 swaps are required to change $A B C$ to $C B A$; one such sequence of swaps is $
A B C \rightarrow B A C \rightarrow B C A \rightarrow C B A .)$

(A) 6
(B) 10
(C) 12
(D) 15
(E) 24

Problem 7

The product of three integers is 60. What is the least possible positive sum of the three integers?

(A) 2
(B) 3
(C) 5
(D) 6
(E) 13

Problem 8

Amy, Bomani, Charlie, and Daria work in a chocolate factory. On Monday Amy, Bomani, and Charlie started working at $1: 00 P M$ and were able to pack 4,3 , and 3 packages, respectively, every 3 minutes. At some later time, Daria joined the group, and Daria was able to pack 5 packages every 4 minutes. Together, they finished packing 450 packages at exactly $2: 45 P M$. At what time did Daria join the group?

(A) $1: 25 \mathrm{PM}$
(B) $1: 35 \mathrm{PM}$
(C) $1: 45 \mathrm{PM}$
(D) 1:55 PM
(E) 2:05 PM

Problem 9

In how many ways can 6 juniors and 6 seniors form 3 disjoint teams of 4 people so that each team has 2 juniors and 2 seniors?

(A) 720
(B) 1350
(C) 2700
(D) 3280
(E) 8100

Problem 10

Consider the following operation. Given a positive integer $n$, if $n$ is a multiple of 3 , then you replace $n$ by $\frac{n}{3}$. If $n$ is not a multiple of 3 , then you replace $n$ by $n+10$. Then continue this process. For example, beginning with $n=4$, this procedure gives $4 \rightarrow 14 \rightarrow 24 \rightarrow 8 \rightarrow 18 \rightarrow 6 \rightarrow$

$ 2 \rightarrow 12 \rightarrow \cdots$. Suppose you start with $n=100$. What value results if you perform this operation exactly 100 times?

(A) 10
(B) 20
(C) 30
(D) 40
(E) 50

Problem 11

How many ordered pairs of integers $(m, n)$ satisfy $\sqrt{n^2-49}=m$ ?

(A) 1
(B) 2
(C) 3
(D) 4
(E) Infinitely many

Problem 12

Zelda played the Adventures of Math game on August 1 and scored 1700 points. She continued to play daily over the next 5 days. The bar chart below shows the daily change in her score compared to the day before. (For example, Zelda's score on August 2 was $1700+80=1780$ points.) What was Zelda's average score in points over the 6 days?

(A) 1700
(B) 1702
(C) 1703
(D) 1713
(E) 1715

Problem 13

Two transformations are said to commute if applying the first followed by the second gives the same result as applying the second followed by the first. Consider these four transformations of the coordinate plane:

Of the 6 pairs of distinct transformations from this list, how many commute?

(A) 1
(B) 2
(C) 3
(D) 4
(E) 5

Problem 14

One side of an equilateral triangle of height 24 lies on line $\ell$. A circle of radius 12 is tangent to line $l$ and is externally tangent to the triangle. The area of the region exterior to the triangle and the circle and bounded by the triangle, the circle, and line $\ell$ can be written as $a \sqrt{b}-c \pi$, where $a$, $b$, and $c$ are positive integers and $b$ is not divisible by the square of any prime. What is $a+b+c$ ?

(A) 72
(B) 73
(C) 74
(D) 75
(E) 76

Problem 15

Let $M$ be the greatest integer such that both $M+1213$ and $M+3773$ are perfect squares. What is the units digit of $M$ ?

(A) 1
(B) 2
(C) 3
(D) 6
(E) 8

Problem 16

All of the rectangles in the figure below, which is drawn to scale, are similar to the enclosing rectangle. Each number represents the area of the rectangle. What is length $A B$ ?

Problem 17

Two teams are in a best-two-out-of-three playoff: the teams will play at most 3 games, and the winner of the playoff is the first team to win 2 games. The first game is played on Team A's home field, and the remaining games are played on Team B's home field. Team A has a $\frac{2}{3}$ chance of winning at home, and its probability of winning when playing away from home is $p$. Outcomes of the games are independent. The probability that Team A wins the playoff is $\frac{1}{2}$. Then $p$ can be written in the form $\frac{1}{2}(m-\sqrt{n})$, where $m$ and $n$ are positive integers. What is $m+n$ ?

(A) 10
(B) 11
(C) 12
(D) 13
(E) 14

Problem 18

There are exactly $K$ positive integers $b$ with $5 \leq b \leq 2024$ such that the base- $b$ integer $2024_b$ is divisible by 16 (where 16 is in base ten). What is the sum of the digits of $K$ ?

(A) 16
(B) 17
(C) 18
(D) 20
(E) 21

Problem 19

The first three terms of a geometric sequence are the integers $a, 720$, and $b$, where $a<720<b$. What is the sum of the digits of the least possible value of $b$ ?

(A) 9
(B) 12
(C) 16
(D) 18
(E) 21

Problem 20

Let $S$ be a subset of ${1,2,3, \ldots, 2024}$ such that the following two conditions hold:

What is the maximum possible number of elements in $S$ ?

(A) 436
(B) 506
(C) 608
(D) 654
(E) 675

Problem 21

The numbers, in order, of each row and the numbers, in order, of each column of a $5 \times 5$ array of integers form an arithmetic progression of length 5 . The numbers in positions $(5,5),(2,4),(4,3)$, and $(3,1)$ are $0,48,16$, and 12 , respectively. What number is in position $(1,2) ?$

(A) 19
(B) 24
(C) 29
(D) 34
(E) 39

Problem 22

Let $\mathcal{K}$ be the kite formed by joining two right triangles with legs 1 and $\sqrt{3}$ along a common hypotenuse. Eight copies of $\mathcal{K}$ are used to form the polygon shown below. What is the area of triangle $\triangle A B C$ ?

(A) $2+3 \sqrt{3}$
(B) $\frac{9}{2} \sqrt{3}$
(C) $\frac{10+8 \sqrt{3}}{3}$
(D) 8
(E) $5 \sqrt{3}$

Problem 23

Integers $a, b$, and $c$ satisfy $a b+c=100, b c+a=87$, and $c a+b=60$. What is $a b+b c+c a$ ?

(A) 212
(B) 247
(C) 258
(D) 276
(E) 284

Problem 24

A bee is moving in three-dimensional space. A fair six-sided die with faces labeled $A^{+}, A^{-}, B^{+}, B^{-}, C^{+}$, and $C^{-}$is rolled. Suppose the bee occupies the point $(a, b, c)$. If the die shows $A^{+}$, then the bee moves to the point $(a+1, b, c)$ and if the die shows $A^{-}$, then the bee moves to the point $(a-1, b, c)$. Analogous moves are made with the other four outcomes. Suppose the bee starts at the point $(0,0,0)$ and the die is rolled four times. What is the probability that the bee traverses four distinct edges of some unit cube?

(A) $\frac{1}{54}$
(B) $\frac{7}{54}$
(C) $\frac{1}{6}$
(D) $\frac{5}{18}$
(E) $\frac{2}{5}$

Problem 25

The figure below shows a dotted grid 8 cells wide and 3 cells tall consisting of $1^{\prime \prime} \times 1^{\prime \prime}$ squares. Carl places 1 -inch toothpicks along some of the sides of the squares to create a closed loop that does not intersect itself. The numbers in the cells indicate the number of sides of that square that are to be covered by toothpicks, and any number of toothpicks are allowed if no number is written. In how many ways can Carl place the toothpicks?

(A) 130
(B) 144
(C) 146
(D) 162
(E) 196

2022 AMC 10A, Problem 20, Hints and Solution

Motivation

To find the last term in a sequence, each term formed by adding similar indexed term from an AP and a GP.

Question

A four-term sequence is formed by adding each term of a four-term arithmetic sequence of positive integers to the corresponding term of a four-term geometric sequence of positive integers. The first three terms of the resulting four-term sequence are 57, 60 and 91. What is the fourth of this sequence?

Hint 1

Use the standard forms of the terms of the progressions to obtain a system of equations.

Hint 2

Try to reduce the number of variables from the system by subtracting two subsequent equations at a time. 

Let $a,ar,ar^2,ar^3$, be the first three terms of the geometric progression, and $b,b+d,b+2d,b+3d$ be the corresponding terms of the arithmetic progression.

We are given, that

$$a+b=57$$

$$ar+b+d=60$$

$$ar^2+b+2d=91.$$

Final Solution

These are 3 non-linear equations in 4 variables, so we can't directly conclude anything. Notice that if we subtract the first two equations we get, discarding $b$ $$3=a(r-1)+d$$ and similarly 

 $$31=ar(r-1)+d.$$

Each of these equations contain, the same variable. So subtracting again, we get

$$28=ar^2-2ar+a=a(r-1)^2.$$

Now since we're dealing with sequences of positive integers, then we can only equate $(r-1)^2$ to either $4$ or $1$.

Then we can conclude that either $a=28$ and $r=2$ or $a=7$ and $r=3$.

If $a=28$, then we get $b=57-28=29$ and $d=-25$. But that makes the arithmetic progression $29,4,-21,-46$, which is a contradiction since the sequence is of positive integers. With $a=7$, $b=50$, and $d=-11$ we get following progressions $50,39,28,17$ and $7,21,63,189$.

The desired number is then

$$17+189=206$$

New AMC 10 & 12 Review Course

Cheenta is offering a 36-hour program on AMC 10 & 12. In this short review course, we will cover concepts from Number Theory, Geometry, Algebra, and Combinatorics. This course is problem-driven in nature, in the sense concepts will be introduced and taught using relevant problems.

Schedule

The program starts on September 9th. The online live class Group A meets on Saturday and Sunday at 6 AM IST. The Group B meets on Saturday and Sunday at 5:30 PM IST. That is we have two time slots. You may choose any one.

Curriculum and Other Details

Apply for Trial Class

AMC 10A 2020 Problem 6 | Divisibility Problem

Try this beautiful Problem based on Divisibility Problem from AMC 2020 Problem 6.

Divisibility Problem: AMC 10A 2020 Problem 6


How many 4-digit positive integers (that is, integers between 1000 and 9999 , inclusive) having only even digits are divisible by 5 ?

Key Concepts


Divisibility

Counting Principle

Suggested Book | Source | Answer


AMC 10A 2020 Problem 6

100

Try with Hints


What is the divisibility rule for a number divisible by 5?

Now apply this for unit, tens, hundred and thousand digits.

Here the unit digit must be 0. So I just have one choice for units place.

The middle two digits can be 0, 2, 4, 6, or 8.

But the thousands digit can only be 2, 4, 6, or 8 since it cannot be zero.

Now try to count how many choices are there for each position.

Then there was 1 choice for unit digit.

5 choices for middle two digits.

4 choices for thousands digit.

Now calculate the total number of choices you can make.

AMC-AIME Program at Cheenta

Subscribe to Cheenta at Youtube


AMC 10A 2021 Problem 9 | Factorizing Problem

Try this beautiful Problem based on Factorizing Problem from AMC 2021 Problem 9.

Factorizing Problem: AMC 10A 2021 Problem 9


What is the least possible value of $(x y-1)^{2}+(x+y)^{2}$ for real numbers $x$ and $y$ ?

Key Concepts


Expansion of Polynomial

Factorization

Suggested Book | Source | Answer


AMC 10A 2021 Problem 9

1

Try with Hints


Expand the expression.

So, we get that the expression is $x^{2}+2 x y+y^{2}+x^{2} y^{2}-2 x y+1$ or $x^{2}+y^{2}+x^{2} y^{2}+1$.

Then the minimum value for this is 1 , which can be achieved at $x=y=0$. .

AMC-AIME Program at Cheenta

Subscribe to Cheenta at Youtube


AMC 10A 2021 I Problem 20 | Enumeration

Try this beautiful Problem based on Enumeration appeared in AMC 10A 2021, Problem 20.

AMC 10A 2021 I Problem 20


In how many ways can the sequence $1$, $2$, $3$, $4$, $5$ be rearranged so that no three consecutive terms are increasing and no three consecutive terms are decreasing?

Key Concepts


Permutation

Enumeration

Combinatorics

Suggested Book | Source | Answer


An Excursion in Mathematics

AMC 10A 2021 Problem 20

32

Try with Hints


We have 5 numbers with us.

So, how many permutations we can have with those numbers?

So, $5!=120$ numbers can be made out of those $5$ numbers.

Now we have to remember that we are restricted with the following condition -

no three consecutive terms are increasing and no three consecutive terms are decreasing.

Now make a list of the numbers which are satisfying the condition given among all $120$ numbers we can have.

Now the list should be -

$13254$, $14253$, $14352$, $15243$, $15342$, $21435$, $21534$, $23154$, $24153$, $24351$, $25143$, $25341$
$31425$, $31524$, $32415$, $32514$, $34152$, $34251$, $35142$, $35241$, $41325$, $41523$, $42315$, $42513$,
$43512$, $45132$, $45231$, $51324$, $51423$, $52314$, $52413$, $53412$.

Count how many permutations are there?

Subscribe to Cheenta at Youtube


Pigeonhole Principle

“The Pigeonhole principle” ~ Students who have never heard may think that it is a joke. The pigeonhole principle is one of the simplest but most useful ideas in mathematics. Let’s learn the Pigeonhole Principle with some applications.

Pigeonhole Principle Definition:

In Discrete Mathematics, the pigeonhole principle states that if we must put $N + 1$ or more pigeons into N Pigeon Holes, then some pigeonholes must contain two or more pigeons.

Example:

If $Kn+ 1$ (where k is a positive integer) pigeons are distributed among n holes than some hole contains at least $k + 1$ pigeons.

Applications of Pigeonhole Principle:

This principle is applicable in many fields like Number Theory, Probability, Algorithms, Geometry, etc.

Problems and Solutions:

Problem 1

A bag contains beads of two colours: black and white. What is the smallest number of beads which must be drawn from the bag, without looking so that among these beads, two are of the same colour?

Solution: We can draw three beads from bags. If there were no more than one bead of each colour among these, then there would be no more than two beads altogether. This is obvious and contradicts the fact that we have chosen their beads. On the other hand, it is clear that choosing two beads is not enough. Here the beads play the role of pigeons, and the colours (black and white) play the role of pigeonhole.

Problem 2

Find the minimum number of students in a class such that three of them are born in the same month?

Solution: Number of month $n =12$

According to the given condition,

$K+1 = 3$

$K = 2$

$M = kn +1 = 2×12 + 1 = 25$.

Problem 3

Show that from any three integers, one can always choose two so that $a^3$b – a$b^3$ is divisible by 10.

Solution: We can factories the term $a^3$b – a$b^3$ = $ab(a + b)(a - b)$, which is always even, irrespective of the pair of integers we choose.

If one of three integers from the above factors is in the form of 5k, which is a multiple of 5, then our result is proved.

If none of the integers is a multiple of 5 then the chosen integers should be in the form of $(5k)+-(1)$ and $(5k)+-(2)$ respectively.

Clearly, two of these three numbers in the above factors from the given expression should lie in one of the above two, which follows by the virtue of this principle.

These two integers are the ones such that their sum and difference is always divisible by 5. Hence, our result is proved.

Problem 4

If n is a positive integer not divisible by 2 or 5 then n has a multiple made up of 1's.

Problem 5

Let $X \subseteq{1,2, \ldots, 99}$ and $|X|=10$. Show that it is possible to select two disjoint nonempty proper subsets $Y, Z$ of $X$ such that $\sum(y \mid y \in Y)=\sum(z \mid z \in Z)$.

Problem 6

Let $A_{1} B_{1} C_{1} D_{1} E_{1}$ be a regular pentagon. For $2 \leq n \leq 11$,
let $A_{n} B_{n} C_{n} D_{n} E_{n}$ be the pentagon whose vertices are the midpoints of the sides of the pentagon $A_{n-1} B_{n-1} C_{n-1} D_{n-1} E_{n-1}$. All the 5 vertices of each of the 11 pentagons are arbitrarily coloured red or blue. Prove that four points among these 55 points have the same colour and form the vertices of a cyclic quadrilateral.

Some Useful links:

External Tangent | AMC 10A, 2018 | Problem 15

Try this beautiful Problem on Geometry based on External Tangent from AMC 10 A, 2018. You may use sequential hints to solve the problem.

External Tangent - AMC-10A, 2018- Problem 15


Two circles of radius 5 are externally tangent to each other and are internally tangent to a circle of radius 13 at points $A$ and $B$, as shown in the diagram. The distance $A B$ can be written in the form $\frac{m}{n}$, where $m$ and $n$ are relatively prime positive integers. What is $m+n ?$

,

  • $21$
  • $29$
  • $58$
  • $69$
  • $93$

Key Concepts

Geometry

Triangle

Pythagoras

Suggested Book | Source | Answer

Suggested Reading

Pre College Mathematics

Source of the problem

AMC-10A, 2018 Problem-15

Check the answer here, but try the problem first

$69$

Try with Hints

First Hint

Given that two circles of radius 5 are externally tangent to each other and are internally tangent to a circle of radius 13 at points $A$ and $B$. we have to find out the length \(AB\).

Now join \(A\) & \(B\) and the points \(Y\) & \(Z\). If we can show that \(\triangle XYZ \sim \triangle XAB\) then we can find out the length of \(AB\).

Now can you finish the problem?

Second Hint

now the length of \(YZ=5+5=10\) (as the length of the radius of smaller circle is $5$) and \(XY=XA-AY=13-5=8\). Now \(YZ|| AB\).therefore we can say that \(\triangle XYZ \sim \triangle XAB\). therefore we can write $\frac{X Y}{X A}=\frac{Y Z}{A B}$

Now Can you finish the Problem?

Third Hint

From the relation we can say that $\frac{X Y}{X A}=\frac{Y Z}{A B}$

\(\Rightarrow \frac{8}{13}=\frac{10}{AB}\)

\(\Rightarrow AB=\frac{13\times 10}{8}\)

\(\Rightarrow AB=\frac{65}{4}\) which is equal to \(\frac{m}{n}\)

Therefore \(m+n=65+4=69\)

Subscribe to Cheenta at Youtube